Intégrale, encore et toujours.

13»

Réponses

  • Je ne pense pas que tu l'as déjà faite en respectant le cahier de charge, c'est le but !
    Le 😄 Farceur


  • Gebrane: je ne sais pas si c'est possible, cela demande réflexion.
  • C'est pourquoi j'ai dit donnons donnons
    Tu réfléchies à la mienne
    Je réfléchies à la tienne
    :-D
    Le 😄 Farceur


  • Je pense avoir établi un lien entre les deux intégrales. Mais il est trop tard (trop tôt) pour que je me recopie mes calculs (et dans lesquels il peut y avoir des erreurs).
  • Je croyais avoir établi une relation entre les deux intégrales mais, en fait, on obtient beaucoup mieux que ça:

    \begin{align}J&=\int_0^1 \dfrac{\ln^2 x\ln(1-x)}{1-x}\,dx\\
    K&=\int_0^1 \frac{\ln^3 \left(\frac{x}{1-x}\right)+\ln^3(1-x)}{1-x} \,dx
    \end{align}Puisque $(a-b)^3+b^3=a^3-3a^2b+3ab^2$ on a:
    \begin{align}K&=\int_0^1 \frac{\ln^3 x}{1-x}\,dx-3J+3\int_0^1 \frac{\ln x\ln^2(1-x)}{1-x}\,dx\end{align}
    Dans la dernière intégrale on applique le changement de variable $y=1-x$,
    \begin{align}K&=\int_0^1 \frac{\ln^3 x}{1-x}\,dx-3J+3\int_0^1 \frac{\ln (1-x)\ln^2 x}{x}\,dx\\
    &=\int_0^1 \frac{\ln^3 x}{1-x}\,dx-3J+\Big[\ln^3 x\ln(1-x)\Big]_0^1+\int_0^1 \frac{\ln^3 x }{1-x}\,dx\\
    &=2\int_0^1 \frac{\ln^3 x}{1-x}\,dx-3J\end{align}Dans l'intégrale qui définit $K$ on fait le changement de variable $y=\dfrac{x}{1-x}$,
    \begin{align}K&=\int_0^\infty \frac{\ln^3 x-\ln^3 (1+x)}{1+x}\,dx\\
    &=\left(\int_0^1 \frac{\ln^3 x}{1+x}\,dx-\int_0^1 \frac{\ln^3 (1+x)}{1+x}\,dx\right)+\int_1^\infty \frac{\ln^3 x-\ln^3 (1+x)}{1+x}\,dx\end{align}Dans la dernière intégrale on applique le changement de variable $y=\dfrac{1}{x}$,
    \begin{align}K&=\int_0^1 \frac{\ln^3 x}{1+x}\,dx-\frac{1}{4}\ln^4 2-\int_0^1 \frac{\ln^3 x+\ln^3\left(\frac{1+x}{x}\right)}{x(1+x)}\,dx\\
    &=\int_0^1 \frac{\ln^3 x}{1+x}\,dx-\frac{1}{4}\ln^4 2-\int_0^1 \frac{\ln(1+x)^3-3\ln^2\left(1+x\right)\ln x+3\ln(1+x)\ln^2 x}{x(1+x)}\,dx\\
    &=\int_0^1 \frac{\ln^3 x}{1+x}\,dx-\frac{1}{4}\ln^4 2+\int_0^1 \frac{\ln^3(1+x)}{1+x}\,dx-\int_0^1 \frac{\ln^3(1+x)}{x}\,dx-3\int_0^1 \frac{\ln^2(1+x)\ln x}{1+x}\,dx+3\int_0^1 \frac{\ln^2(1+x)\ln x}{x}\,dx-\\
    &3\int_0^1 \frac{\ln(1+x)\ln^2 x}{x}\,dx+3\int_0^1 \frac{\ln(1+x)\ln^2 x}{1+x}\,dx\\
    &=\int_0^1 \frac{\ln^3 x}{1+x}\,dx-\int_0^1 \frac{\ln^3(1+x)}{x}\,dx-3\int_0^1 \frac{\ln^2(1+x)\ln x}{1+x}\,dx+3\int_0^1 \frac{\ln^2(1+x)\ln x}{x}\,dx-3\int_0^1 \frac{\ln(1+x)\ln^2 x}{x}\,dx+\\
    &3\int_0^1 \frac{\ln(1+x)\ln^2 x}{1+x}\,dx
    \end{align}
    Hormis pour les première et seconde intégrales, et la dernière intégrale de la dernière ligne de calcul on effectue une intégration par parties:
    \begin{align}K&=\int_0^1 \frac{\ln^3 x}{1+x}\,dx-\int_0^1 \frac{\ln^3(1+x)}{x}\,dx+\int_0^1 \frac{\ln^3(1+x)}{x}\,dx-3\int_0^1 \frac{\ln(1+x)\ln^2 x}{1+x}\,dx+\int_0^1 \frac{\ln^3 x}{1+x}\,dx+\\
    &3\int_0^1 \frac{\ln(1+x)\ln^2 x}{1+x}\,dx\\
    &=2\int_0^1 \frac{\ln^3 x}{1+x}\,dx\\
    &=2\int_0^1 \frac{\ln^3 x}{1-x}\,dx-2\int_0^1 \frac{2x\ln^3 x}{1-x^2}\,dx\\
    \end{align}Dans la deuxième intégrale on applique le changement de variable $y=x^2$:
    \begin{align}K&=2\int_0^1 \frac{\ln^3 x}{1-x}\,dx-\frac{1}{4}\int_0^1 \frac{\ln^3 x}{1-x}\,dx\\
    &=\frac{7}{4}\int_0^1 \frac{\ln^3 x}{1-x}\,dx
    \end{align}$J$ vérifie donc l'équation: $\displaystyle \frac{7}{4}\int_0^1 \frac{\ln^3 x}{1-x}\,dx=2\int_0^1 \frac{\ln^3 x}{1-x}\,dx-3J$ Ainsi $\boxed{\displaystyle J=\frac{1}{12}\int_0^1 \frac{\ln^3 x}{1-x}\,dx}$.
    Si on admet que $\displaystyle \int_0^1 \frac{\ln^3 x}{1-x}\,dx=-6\zeta(4)$ alors $\boxed{\displaystyle J=-\frac{1}{2}\zeta(4)}$

    PS:
    Gebrane: c'était un marché de dupes, tu récupères l'intégrale la plus difficile à calculer. B-)
    (cela dit, tu peux deviner la méthode à laquelle je pensais maintenant)
  • waw, je suis émerveillé, je vais te relire minutieusement en face à la mer.
    C'est un monde magique "les intégrales"
    Le 😄 Farceur


  • WAWO $Fin \; de \; partie$ Samouraï avec deux épées en combats. Mes respects89120
  • FDP
    Avec le nouveau $$K=\int_0^1 \frac{\ln^3 \big(\frac{x}{1+x}\big)+\ln^3(1+x)}{1+x} \,dx$$ l’intégrale difficile tombe ?
    Le 😄 Farceur


  • Gebrane:
    Pour calculer $\displaystyle \int_0^1 \frac{\ln^2 x\ln(1+x)}{1+x}\,dx$ je te conseille plutôt de considérer cette intégrale $\displaystyle \int_0^1 \frac{\ln^3\left(\frac{x}{x+1}\right)}{1+x}\,dx$

    Je n'ai pas mené les calculs jusqu'au bout mais je pense qu'on y arrive mais on a besoin de la formule rappelée plus haut pour faire apparaître des polylogarithmes.
  • Ou, peut-être plus simplement, on part de l'intégrale $\displaystyle \int_0^1 \frac{\ln^2 x\ln(1-x)}{1-x}\,dx$ et on applique le changement de variable adéquat pour faire apparaître une expression qui contient $\displaystyle \int_0^1 \frac{\ln^2 x\ln(1+x)}{1+x}\,dx$
  • FDP ton cahier de charge, nous autorise-il l'utilisation de Li_2 et ses propriétés ?
    Le 😄 Farceur


  • Gebrane: tu peux utiliser, pour $|a|<1$, $r\geq 2$,

    \begin{align}\int_0^1 \frac{a\ln^r x}{1-ax}\,dx=(-1)^r r! \text{Li}_{r+1}(a)\end{align}


    Je ne vois pas comment faire d'autre si on prend comme définition $\displaystyle \text{Li}_r(x)=\sum_{n=1}^{\infty} \frac{x^r}{n^r}$ avec $r\geq 2$ et $|x|<1$

    Autrement tu prends comme définition, pour $|a|<1$:

    \begin{align} \text{Li}_{r+1}(a)=\frac{(-1)^r}{ r!}\int_0^1 \frac{a\ln^r x}{1-ax}\,dx\end{align}

    PS:
    Toutes les formules sur les polylogarithmes s'obtiennent probablement par changement de variable et intégration par parties. Tu peux tester pour obtenir la formule qui lie $\text{Li}_2(x)$ et $\text{Li}_2(1-x)$
    En prenant pour définition (ou comme propriété) que $\displaystyle \text{Li}_2(x)=-\int_0^1 \frac{x\ln t}{1-tx}\,dt$
  • Gebrane:
    Ce n'est pas la même intégrale et, en outre, il y a une partie du calcul qui ne répond pas à mon cahier des charges si je lis bien.
  • C la même intégrale, il y a une ipp.
    Le 😄 Farceur


  • Gebrane:

    Comment passes-tu de l'intégrale $\displaystyle \int_0^1 \frac{\ln^2 x\ln(1+x)}{1-x}\,dx$ (l'intégrale dont il est question dans le lien M.E) à l'intégrale $\displaystyle \int_0^1 \frac{\ln^2 x\ln(1+x)}{1+x}\,dx$ par intégration par parties? Je suis curieux de l'entendre.
  • FDP

    C'est de ma faute, j'ai donné le faux lien, le bon ici https://math.stackexchange.com/questions/1341254/about-the-integral-int-01-frac-logx-log21xx-dx/1341866#1341866
    Leur intégrale est l'opposé du notre ( par ipp)
    Le 😄 Farceur


  • En effet, cette solution mise en lien a l'air bien. Elle utilise quelques propriétés sur la dérivée des fonctions polylogarithmes. L'approche n'est pas si différente de celle que j'ai en vue.
  • tu as résolu la mienne, j'ai "résolu" la tienne
    A un prochain donnons donnons :-D
    Le 😄 Farceur


  • J'aimerais bien réussir à montrer, avec le même cahier des charges que:

    \begin{align}J&=\int_0^1 \frac{\ln^3 x\ln(1-x)}{1-x}\,dx=12\zeta(5)-6\zeta(2)\zeta(3)\end{align}

    La présence du terme $\zeta(2)\zeta(3)$ semble être un frein à cette prétention mais en même temps (sors de ce corps Emmanuel (:D ) cela donne l'idée de pratiquer une forme de "reverse engineering".

    En effet, $\displaystyle \zeta(2)\zeta(3)=\Big[\text{Li}_2(x)\text{Li}_3(x)\Big]_0^1=-\int_0^1 \frac{\ln(1-x)\text{Li}_3(x)}{x}\,dx+\int_0^1 \frac{\text{Li}^2_2(x)}{x}\,dx$
    En continuant le calcul on peut faire apparaître l'intégrale à calculer.
    La question initiale devient de montrer qu'une certaine intégrale est nulle. Peut-être que la nouvelle est plus simple à calculer je l'ignore à cette heure.

    NB:\begin{align}\frac{\partial}{\partial x}\text{Li}_2(x)&=-\frac{\ln(1-x)}{x}\\
    \frac{\partial}{\partial x}\text{Li}_3(x)&=\frac{\text{Li}_2(x)}{x}
    \end{align}
  • Bien reçu
    Le 😄 Farceur


  • On peut écrire l’intégrale avec la fonction Beta d'Euler, c'est interdit?
    Le 😄 Farceur


  • Gebrane:

    Oui, c'est interdit, tu vas dériver (deux fois) une fonction Bêta à paramètres.
  • Je n'ai pas progressé dans le calcul de la précédente intégrale.

    Je vous propose néanmoins de montrer, par intégration par parties et changement de variable (sans intégrales doubles) que:\begin{align}\int_0^1 \frac{\ln^3\left(\frac{x}{1-x}\right)}{1+x}\,dx=-\frac{1}{4}\ln^4 2-\frac{1}{2}\pi^2\ln^2 2\end{align}
    Je pense que ce calcul est possible, bien que je ne l'ai pas encore mené à son terme en respectant le cahier des charges.

    NB:
    On admet aussi que \begin{align}\int_0^1 \frac{\ln x}{1-x}\,dx=-\frac{\pi^2}{6}\end{align}

    PS: Cette intégrale est bien calculable avec ce cahier des charges. (Calculs effectués)
  • Peux-tu la poser sur ME pour voir
    De ma part, avec cette chaleur je ne peux me concentrer plus de 5 mn
    Le 😄 Farceur


  • Gebrane:
    Ce calcul s'inscrit dans une tentative de calculer:
    \begin{align}\int_0^1 \frac{\ln x\ln^2(1-x)}{1+x}\,dx\end{align}

    ( https://math.stackexchange.com/questions/3325606/how-to-evaluate-int-01-frac-ln-x-ln21-x1x-dx-in-an-elegant-way )

    Mais pour le moment je n'avance guère plus que ce bon début.
  • FDP Si j'ai bien compris, tu cherches à calculer ( avec ton cahier de charge )

    $$\int_0^1 \frac{\ln x\ln^2(1-x)}{1+x}\,dx$$

    D’après le lien que tu as mentionné https://math.stackexchange.com/questions/3325606/how-to-evaluate-int-01-frac-ln-x-ln21-x1x-dx-in-an-elegant-way , Zacky a besoin de calculer $$Q=\int_0^1 \frac{\ln(1+x)\ln^2 x}{1+x}dx$$
    Mais, on en a deja parlé et on sait le calculer d’après

    https://math.stackexchange.com/questions/1341254/about-the-integral-int-01-frac-logx-log21xx-dx/1341866#1341866
    Le 😄 Farceur


  • Gebrane:
    Zacky utilise aussi une, ou plusieurs, intégrales Beta d'Euler qu'il dérive

    Pour finir le calcul il faudrait pouvoir calculer $\displaystyle \int_0^1 \frac{\ln^3(x(1-x))}{1+x}\,dx$.*
    Je ne suis pas certain d'y parvenir et j'ai essayé une tonne de trucs de Sioux. Un va peut-être fonctionner mais je n'en suis pas sûr.


    *: Empiriquement je connais la réponse, c'est une combinaison linéaire à coefficients rationnels des constantes: $\displaystyle \text{Li}_4\left(\frac{1}{2}\right),\zeta(4),\ln^4 2,\pi^2\ln^2 2$
  • En suivant la piste des Sioux (mathématiciens), toujours en rapport avec le calcul précédent je me retrouve confronté à une intégrale de la forme suivante:
    \begin{align} \int_0^1 \frac{(1-x)\ln^2\left(\frac{\sqrt{x}}{1+x}\right)R(x)}{x(1+x)}\,dx\end{align}
    $R$ est une fonction à dérivée continue sur $[0;1]$ et telle que $\lim_{x\rightarrow 0}R(x)\ln x=0$.
    Il me semble que cette intégrale a des propriétés intéressantes (en vue de son calcul, il y a des simplifications qui sont possibles et que je considère comme étonnantes).

    PS:
    Dans le calcul qui m'intéresse, mais cela n'intervient pas dans la simplification que je crois voir, la fonction $R$ est le logarithme d'une fraction rationnelle (le dénominateur et le numérateur sont des fonctions affines).
  • La formule dont je parlais plus haut avec $R$ une fonction à dérivée continue telle que:
    $\displaystyle \lim_{x\rightarrow 0}R(x)\ln x=\lim_{x\rightarrow 0}R(x)\ln^2 x=\lim_{x\rightarrow 0}R(x)\ln^3 x=0$
    On a:\begin{align}\int_0^1 \frac{(1-x)\ln^2\left(\frac{\sqrt{x}}{1+x}\right)R(x)}{x(1+x)}\,dx
    &=-\frac{1}{12}\int_0^1 R^\prime(x)\ln^3 x\,dx+\frac{2}{3}\int_0^1 R^\prime(x)\ln^3(1+x)\,dx+\frac{1}{2}\int_0^1 R^\prime(x)\ln^2 x\ln(1+x)dx-\\
    &\int_0^1 R^\prime(x)\ln x\ln^2(1+x)dx-\frac{2}{3}R(1)\ln^3 2\\
    &=-\frac{1}{12}\int_0^1 R^\prime(x)\ln^3 x\,dx+\frac{1}{6}\int_0^1 R^\prime(x)\ln^3(1+x)\,dx+\frac{1}{2}\int_0^1 R^\prime(x)\ln(1+x)\ln^2\left(\frac{x}{1+x}\right)\,dx-\\
    &\frac{2}{3}R(1)\ln^3 2\\
    \end{align}

    Il faut penser que $R^\prime$ est une fraction rationnelle du type: $\dfrac{d}{(ax+b)(a'x+b')}$
    Ce qui fait que les deux premiers termes de la dernière ligne devraient être exprimables "aisément" à base de polylogarithmes et que le seul point noir est la dernière intégrale. A cette heure, j'ignore si cette formule a une quelconque utilité dans le calcul que j'essaie de terminer mais je la trouve jolie tout de même. B-)-
  • Je vous propose de montrer que : $$
    \int_{\frac{1}{\sqrt{3}}}^1 \frac{\arctan\left(\sqrt{2-x^2}\right)}{1+x^2}\,dx=\frac{11\pi^2}{480}

    $$ Je ne sais pas encore la calculer. Je vous avertis que, si mes yeux ne me jouent pas des tours, cette intégrale est encore plus retorse qu'elle y parait. Quand on connaît un peu ce type d'intégrales on a envie de se jeter sur une méthode mais celle-ci semble échouer dans le cas d'espèce.

    PS. En fait, très curieusement, je me retrouve dans une situation très comparable face à cette intégrale et face à la précédente intégrale. :-D
Connectez-vous ou Inscrivez-vous pour répondre.